FP511 Practice Exam

Réussis tes devoirs et examens dès maintenant avec Quizwiz!

Choose the duties that must be fulfilled for a CFP® professional to act as a fiduciary. Duty of Loyalty Duty of Care Duty to Follow Client Instructions

all At all times when providing Financial Advice to a Client, a CFP® professional must act as a fiduciary, and therefore, act in the best interests of the Client. The following duties must be fulfilled: 1) duty of loyalty, 2) duty of care, and 3) duty to follow client information.

Which one of the following should be considered a liquid asset for emergency fund purposes?

savings accounts. Savings accounts may be accessed quickly with no loss of capital. Life insurance cash values, stock mutual funds, and personal residences usually cannot.

Today, Charlotte purchased an oil painting for $75,000. She expects the painting's value to increase at a rate of 11%, compounded annually for the next seven years. How much will the painting be worth at the end of the seventh year if her expectations are correct?

$155,712.01. This is a future value of a single sum calculation. The keystrokes on the HP 10bII/HP 10bII+ are as follows: END mode 1, DOWNSHIFT, P/YR DOWNSHIFT, C ALL 75000, +/-, PV 7, DOWNSHIFT, N 11, I/YR Solve for FV = 155,712.0115, or $155,712.01.

You have gathered the following information from Edgar's financial statements: Net income $75,000 Gross income $110,000 Total assets $190,000 Total debt $45,000 Consumer debt $20,000 Based on this information, which of the following statements is CORRECT? 1 Edgar's total debt ratio exceeds the generally recommended maximum. 2 Edgar's consumer debt ratio exceeds the generally recommended maximum.

all It is generally recommended that total debts do not exceed 36% of gross income. Edgar's total debt ratio is 40.9%, greater than the 36% maximum ($45,000 / $110,000 = 40.9%). The consumer debt ratio is the ratio of consumer debt payments to net income. Edgar's consumer debt ratio is 26.67%, which exceeds the generally recommended maximum of 20% ($20,000 / $75,000 = 26.67%)

What would the inflation-adjusted interest rate be with a 2.5 % inflation rate and a 6% rate of return? Round to two decimal places.

3.41% [(1.06 ÷ 1.025) − 1] x 100 = 3.4146, rounded to 3.41 Note: Alternate calculation would be 1.025, INPUT 1.06, DOWNSHIFT, % CHG Solve for I/YR = 3.41%

At Harper's 15th birthday party today, Sam and Linda give Harper an audio interface, and a promise that on her 17th birthday, they will pay her tuition for after-school recording lessons. The tuition for the recording school is expected to be $3,000. How much should Sam and Linda invest on Harper's birthday this year at an annual interest rate of 5.5% compounded annually to fulfill their promise?

$2,695. This is a present value of a single sum calculation. Harper is 15 years old on her birthday this year; her 17th birthday is in two years. Keystrokes on the HP 10bII/HP 10bII+ are as follows: END Mode 1, DOWNSHIFT, P/YR DOWNSHIFT, C ALL 3000, FV 5.5, I/YR 2, DOWNSHIFT, N Solve for PV = −2,695.3572 or $2,695 (rounded).

(Martin Case Study) Calculate the Martins' savings from refinancing their home loan for the first month of the new loan. Assume they refinance with a 30-year loan. The result is

$643.06. Current monthly mortgage payment—$1,829.50 Mortgage payment after refinancing—$1,186.44 Monthly savings—$643.06 (Do not factor in closing costs because they are being paid from separate funds.)

Which of the following statements regarding verbal mirroring is CORRECT? 1 The use of verbal mirroring can improve rapport with clients. 2 In verbal mirroring, the planner imitates the client's word use, tone of voice, and communication method. 3 In verbal mirroring, the planner uses the client's body language. 4 Verbal mirroring includes the inflection of voice or emphasis on certain words.

1,2 Statement III is incorrect because the use of the client's body language is physical mirroring. Statement IV is incorrect because voice tone is the inflection of voice or emphasis on certain words.

Which of the following are state law exemptions found in Chapter 7 of the Federal Bankruptcy Code? 1 Disability income benefits 2 An exemption for one's homestead 3 Property held as tenants by the entirety 4 Existing cash value of life insurance policies

all Other state law exemptions include the proceeds of any annuity contracts.

*** Henry's son, Brock, turned six years old today. Henry has plans for Brock to attend a four-year private university at age 18. Currently, tuition is $46,500 per year and is expected to increase at 6.5% per year. Henry can earn an annual compound investment return of 9%. Calculate the lump sum that he needs on Brock's first day of college to be able to pay for his entire college education.

$382,594.77. Step 1 Determine the future cost of college for the first year: 46,500 +/- PV 6.5 I/YR 12 N Solve for FV = 99,002.9752, or $99,002.98 Step 2 Determine the account balance necessary to fund college education: BEG mode (money is needed at the beginning of college) 99,002.9752 +/- PMT 2.3474 I/YR [(1.09 ÷ 1.065) - 1] × 100 = 2.3474 4 N Solve for PV = 382,594.7657, or $382,594.77

Which of the following are considered fixed cash outflows? 1 Clothing expenses 2 Mortgage payments 3 Insurance premiums 4 Auto loan payments

2,3,4 Clothing expenses are a variable outflow because they typically do not occur on a regular basis and the amount tends to vary. The other choices represent fixed outflows because they tend to occur regularly and the amount is more predictable.

According to the rules established by CFP Board, which of the following uses of the certification marks are CORRECT? 1 Randolph Jones, CFP® 2 Randolph Jones, C.F.P. 3 Randolph Jones & Co., PA, CFPs 4 Randolph Jones, CERTIFIED FINANCIAL PLANNER™

1,4 The CFP® marks should never contain periods. In addition, the marks should not be used as part of or incorporated in the name of a firm.

Which of the following are characteristics of Chapter 13 bankruptcy provisions? 1 A repayment plan is implemented. 2 The debtor is typically required to relinquish assets. 3 Debt payments may be reduced so payments are more manageable for the debtor. 4 Generally, Chapter 13 bankruptcy is more favorable for creditors than Chapter 7 bankruptcy.

1,3,4 Chapter 13 bankruptcy is generally more favorable for creditors because they receive at least some portion of what is owed. In a Chapter 13 bankruptcy, the debtor repays at least a portion of the debts over a specified time. Creditors may not receive any payments under a Chapter 7 bankruptcy. In a Chapter 13 bankruptcy, debtors are typically not required to relinquish assets.

(Martin Case Study) Ryan and Sherri are considering refinancing their current mortgage loan. What is the monthly payment for a 30-year loan refinance?

$1,186.44. END Mode 12, DOWNSHIFT, P/YR C ALL 30, DOWNSHIFT, N 6, I/YR (Interest rate is 6% for refinancing—from case facts; 360 periods show on display) 197,888, PV (Mortgage balance—from Statement of Financial Position) FV = 0 Solve for PMT = −1,186.4385, or $1,186.44

If you are a registered investment adviser under the Investment Advisers Act of 1940, which of the following situations constitutes a violation of this act?

You provide the written disclosure brochure to your client at the end of the 3rd day following the engagement letter. Under the brochure rule of this act, you must deliver the brochure (Form ADV Part 2A) at least 48 hours before entering into any written or oral contract with a client.

Which of the following are functions of banks? Provide safe deposit boxes Pay interest on deposits Issue cashier's checks Accept deposits

all

Which of the following statements regarding stagflation is CORRECT? Inflation and unemployment rise. The general growth of the economy is slow as business output falls.

both Stagflation occurs when inflation and unemployment rise and the general growth of the economy is slow as business output falls.

Aaron, a financial planner, has advised Macy that she needs to increase her savings. What might you recommend as a good savings strategy? 1 Limit the number of lattes she purchases. 2 Use an overdraft feature on debit cards. 3 Increase her deductible on her car insurance policy. 4 Limit her credit card purchases to those she can pay off in full in one year.

1,3 Macy can use the money she saves by limiting the number of lattes she purchases to increase her savings. Using an overdraft feature on debit cards may tempt her to spend money she does not have available in her account. Increasing insurance deductibles decreases premiums, which is a good savings strategy. If credit cards are used, they should be paid off in full at the end of each month.

What sanction represents a period of time in which Respondent remains subject to the Terms and Conditions of Certification and Trademark License, but is not certified or Licensed and is prohibited from using the CFP® certification marks?

Suspension. A Suspension is a period of time in which Respondent remains subject to the Terms and Conditions of Certification and Trademark License but is not certified or licensed and is prohibited from using the CFP® certification marks.

What would the inflation-adjusted interest rate be with a 5.5% rate of return and a 3% inflation rate? (Rounded to two decimal places.)

2.43%. [(1.055 ÷ 1.03) - 1] x 100 = 2.4272, [2.43, rounded]

Which of the following individuals are required to register as an investment adviser under the Investment Advisers Act of 1940? 1 Bethany provides advice regarding securities. 2 Chase receives compensation for providing advice. 3 Andrew is in the business of providing advice about securities. 4 Catherine is a CFP® practitioner who issues reports regarding securities.

All

Which of the following statements regarding supply and demand is CORRECT? As prices change, both quantity demanded and quantity supplied respond accordingly. Prices should always move toward equilibrium unless restricted by outside sources, such as government regulation.

both

Rachelle's daughter, Sara, will attend college 10 years from today. She will need $15,000 at the beginning of each academic year for four years. This cost has been guaranteed and will not increase in the period before or during college. What amount will Rachelle need to deposit at the beginning of each month, starting now, to fund Sara's college education if an 8% annual rate of return (compounded monthly) can be earned on these funds? Assume deposits into the education fund stop when Sara starts college.

$291.35. This requires a present value of an annuity due calculation. First, calculate the present value of total savings needed on a lump-sum basis as follows: BEG Mode 1, DOWNSHIFT, P/YR DOWNSHIFT, C ALL 8, I/YR 15000, +/−, PMT 4, DOWNSHIFT, N Solve for PV of 53,656.45 Next, use the PV as an FV and discount back to calculate the monthly payment: BEG Mode 12, DOWNSHIFT, P/YR DOWNSHIFT, C ALL 10, DOWNSHIFT, N = 120 8, I/YR 53,656.45, FV Solve for PMT = −291.3488, or $291.35. 8.1.1

Mickey has decided he needs to increase the balance of his emergency fund. Which of the following are ways Mickey can save for this purpose? 1 Cancel his audiobook subscription. 2 Choose a more economical cell phone plan. 3 Use an overdraft feature on his debit cards. 4 Decrease the deductible on his automobile insurance policy.

1-2 Using an overdraft feature on debit cards may entice Mickey to spend money he does not have available in his account. Decreasing insurance deductibles increases premiums, which is not a savings strategy.

*** When providing financial planning services to a client, a certificant needs to provide these services with integrity. Which of the following are characteristics of the duty of Integrity? 1 Integrity ensures that information is accessible only to those authorized to have access. 2 Integrity demands honesty and candor, which must not be subordinated to personal gain an advantage. 3 Allowance can be made for innocent error and legitimate differences of opinion, but integrity cannot coexist with deceit or subordination of one's principles.

2-3 The duty of confidentiality and privacy ensures that information is accessible only to those authorized to have access.

Which of the following are state law exemptions found in Chapter 7 of the Federal Bankruptcy Code? 1 Pension and retirement plan rights (ERISA plans) 2 A limited amount of personal property 3 Proceeds from annuity contracts 4 Disability income benefits

all Other state law exemptions include an exemption for one's homestead and the existing cash value of any life insurance policies.

Which of the following statements regarding body language is CORRECT? A planner's body language has significant impact on how clients receive messages. Body language involves facial expressions, eye contact, gestures, and body posture.

both A planner's body language has considerable impact on how clients receive messages. Body language involves facial expressions, eye contact, gestures, and body posture.

All of the following would be included on a client's statement of financial position except A) the client's credit card balances. B) the value of the client's home. C) the client's salary. D) the client's checking account balance.

c The statement of financial position shows the client's assets, liabilities, and net worth. The client's salary is a cash inflow and would be included on the statement of cash flows, not the statement of financial position.

Zach expects to receive $20,000 in eight years from his uncle Bradley. How much is this worth in today's dollars assuming a 6% after-tax interest rate, compounded annually?

$12,548. This requires a present value of a single sum calculation. The keystrokes on the HP 10bII/HP 10bII+ are as follows: END Mode 1, DOWNSHIFT, P/YR DOWNSHIFT, C ALL 20000, FV 6, I/YR 8, DOWNSHIFT, N Solve for PV = −12,548.2474, or $12,548.

Which of the following statements describe characteristics of deflation? 1 Prices are falling in absolute terms. 2 The rate of inflation has declined.

1 Disinflation, not deflation, is characterized by a decline in the rate of inflation. Deflation is characterized by prices that are falling in absolute terms.

Which of the following statements concerning the Consumer Credit Protection Act is CORRECT? 1 Credit terms must be disclosed before extending credit. 2 Consumer liability for a lost or stolen credit card is limited to $100. 3 Interest must be reported in terms of annual percentage rate (APR). 4 Credit bureau reports must include accurate, relevant, and recent information.

1,3 Statement II is incorrect because a cardholder is liable for only $50 in unauthorized charges if he reports the card as lost or stolen. Statement IV is incorrect as it is a requirement of the Consumer Credit Reporting Act, not the Consumer Credit Protection Act. The Consumer Credit Protection Act also requires that applicants who are denied credit must be offered the reason

Johanna is considering purchasing a greenhouse for use in her business that will cost $25,000. She anticipates selling this machine at the end of five years for $7,500. The machine is projected to produce the following cash flows: End of year 1: $500 End of year 2: $750 End of year 3: $1,000 End of year 4: $2,100 End of year 5: $4,300 Calculate the net present value (NPV) of purchasing the machine if Johanna's opportunity cost is 10%.

−$14,413.11. Keystrokes on the HP 10bII/HP 10bII+ are as follows: 25000, +/−, CFj 500, CFj 750, CFj 1000, CFj 2100, CFj 11800, CFj 10, I/YR DOWNSHIFT, NPV = -14,413.1052, or −$14,413.11

Sandy would like to reduce her debt by paying off the debt with the lowest balance first, then paying off the next lowest balance, and so on. She currently has $50 in excess monthly cash flow to start this process. Her debts are listed below. Using the snowball technique, what would she be able to pay on the Card D once the smaller loans have been paid off? Debt - Balance -Minimum Payment Card A - $225 - $25 Card B - $575 - $35 Card C - $1,000 - $50 Card D - $3,200 - $150 Auto loan - $12,000 - $300

$310 Sandy would have $50 initially plus $110 after Cards, A, B, and C are paid off. Add that to the $150 she has been paying on Card D for a total of $310. ($50 + $110 + $150 = $310)

Stuart needs an income stream equivalent to $50,000 in today's dollars at the beginning of each year for the next 12 years to maintain his standard of living. He assumes inflation will average 4.5% over the long term and he can earn a 9% compound annual after-tax rate of return on investments. What lump sum does Stuart need to invest today to fund his income need?

$480,878.04. BEG Mode 1, DOWNSHIFT, P/YR DOWNSHIFT, C ALL 50000, PMT [(1.09 ÷ 1.045) − 1] × 100 = 4.3062, I/YR 12, DOWNSHIFT, N 0, FV Solve for PV = -480,878.04, or $480,878.04

The preamble of the Code of Ethics and Standards of Conduct describes the value of upholding aspirational standards and incorporating them into everyday financial planning practice. Identify the purpose(s) of the Code and Standards contained in the preamble. 1 To provide standards for delivering financial planning 2 To advance financial planning as a distinct and valuable profession 3 To hold all CFP® professionals to high standards of competency and ethics 4 To investigate, review, consider recommendations, and issue final decisions on allegations of violations and/or non-compliance

1-3 Statement IV is incorrect; the CFP Board's Disciplinary and Ethics Commission is responsible for investigating, reviewing, considering recommendations, and issuing final decisions on allegations of violations and/or noncompliance.

Marcel purchased a zero-coupon bond 12 years ago for $300. The bond, with a face value of $1,000, matured today. What is the average annual compound rate of return (calculated semiannually) that Marcel realized on his investment?

10.29%. Marcel averaged 10.29% on his investment, calculated as follows: END Mode 2, DOWNSHIFT, P/YR DOWNSHIFT, C ALL 300, +/-, PV 1000, FV 12, DOWNSHIFT, N = 24 0, PMT Solve for I/YR = 10.29 or 10.29%

(Martin Case Study) Assuming the Martins' original 30-year fixed mortgage loan amount was $200,000, what is the interest rate on the loan?

10.5% END Mode 12, DOWNSHIFT, P/YR C ALL 200,000, PV 30, DOWNSHIFT, N 1.829.50, PMT (Mortgage payments from Statement of Cash Flows ÷ 12) Solve for I/YR = 10.5%

Randy is trying to decide whether he should lease or purchase an automobile for his personal use. Which of the following should be considered as he makes this decision? 1 Itemized income tax deductions that are available. 2 There is interest forgone on the funds used as a down payment if they purchase the vehicle. 3 Leasing an automobile generally costs more in total dollar terms than buying a car for cash. 4 At the end of the lease term, they may have nothing to show for the dollars expended.

2-4 Neither lease payments nor the interest paid for automobile financing is income tax deductible unless the automobile is substantially used for business purposes. The automobile that Randy is purchasing is for his personal use.

Which of these statements regarding the provisions of the Bankruptcy Abuse Prevention and Consumer Protection Act of 2005 (the 2005 Bankruptcy Act) are CORRECT? 1 Debtors who want to file for Chapter 7 or Chapter 13 must first undergo credit counseling. 2 Chapter 7 allows for the liquidation of all credit debt or loans, both secured and unsecured. 3 People who have the ability to pay their debts are required to file under Chapter 13. 4 Lenders must provide consumer information about the dangers of paying only minimum required payments on credit card debt.

3-4 Only debtors who file for Chapter 7 bankruptcy must first undergo credit counseling. Use of Chapter 7 bankruptcy is limited to the liquidation of credit debt or loans that are unsecured.

Which of the following actions would most likely help an individual improve their credit score?

Make the minimum payment on time each month. Although making a minimum payment on a credit card will mean an individual will pay significant interest over the years, it shows creditworthiness and, over time, would increase a credit score.

All of the following are reasons for analyzing a client's cash flow except A) providing a snapshot of the client's net worth on any given date. B) determining the client's ability to save and invest. C) determining whether the client is living within or beyond his means. D) revealing problem areas in the client's spending patterns.

a The statement of financial position, or net worth statement, provides a snapshot of the client's net worth on any given date.

As a CFP professional and the Martins' financial planner, you are required to act as a fiduciary. According to CFP Board, what requirements must you fulfill? 1 Engage the client with care, skill, prudence, and diligence. 2 Consider the client's goals, risk tolerance, objectives, and circumstances. 3 Avoid, fully disclose, obtain consent or manage Material Conflicts of Interest. 4 Adhere to the Terms of the Engagement and follow "reasonable and lawful" client instructions.

all

Which of the following statements regarding financial institutions is CORRECT? A savings and loan association (S&L) is also known as a thrift institution. A credit union, owned by its members, is a financial institution that accepts deposits and makes loans. A brokerage company is an intermediary that facilitates transactions involving sales of investments or real estate. A credit union allocates earnings from loan interest and investments to its members in the form of dividends.

all

Which of the following are measures of the Fair and Accurate Credit Transaction Act (FACTA) that provides consumers greater protection against identity theft? Consumer information must be disposed of by companies in a secure manner. Individuals can place alerts on their credit histories if identity theft is suspected. Consumers can place alerts on their credit histories if deploying overseas in the military. Consumers may obtain a free credit report every 12 months from each of the three national credit reporting agencies.

all All are measures of FACTA, which added new sections to the federal Fair Credit Reporting Act to provide consumers greater protection against identity theft.

Which of the following are Integration Factors that show a Financial Planning Engagement exists? The number of relevant elements of the Client's personal and financial circumstances that the Financial Advice may affect The effect on the Client's overall exposure to risk if the Client implements the Financial Advice

both

Which of the following statements correctly applies the principles found in CFP Board's Code and Standards? Competence means providing professional services with relevant knowledge and skill to apply that knowledge. Integrity demands honesty and candor, which must not be subordinated to personal gain or advantage.

both

All of the following statements regarding the provisions of the Bankruptcy Abuse Prevention and Consumer Protection Act of 2005 are correct except A) the use of Chapter 7 is limited to the liquidation of credit bills or loans that are not secured. B) debtors who want to file for Chapter 7 must first undergo credit counseling. C) people who have the ability to pay their debts can choose between filing under Chapter 7 and filing under Chapter 13. D) lenders must provide consumer information about the dangers of paying only minimum required payments on credit card debt.

c Under the 2005 Bankruptcy Act, people who have the ability to pay their debts must file under Chapter 13 (reorganization) rather than having their debts canceled entirely under Chapter 7.

The Consumer Price Index (CPI) is falling. Consumer sentiment is strong, as are auto and retail sales. Labor productivity is declining. What stage of the business cycle is the economy likely experiencing?

expansion to peak. When the economy is moving from expansion to peak, labor productivity starts to decline, consumer sentiment is strong, and the CPI is falling.

(Martin Case Study) During conversations with Sherri, you see she considers her investment skills to be much greater than they actually are. She tends to take credit for any investment decisions that have positive returns for her family, but blames the economy when their portfolio does poorly. Sherri's behavior is an example of

overconfidence. Sherri's behavior is an example of overconfidence. Confirmation bias is paying attention to information that supports a preconceived opinion and poorly made decision, while disregarding accurate, unsupportive information. Anchoring is making irrational decisions based on information that should have no influence on the decision at hand. Herding is present when individuals tend to follow the actions of a larger group of people when making financial decisions, whether those actions are rational or not.

Bill and Brandy want to set up a program to fund their son's college education. Their son has said he will likely want to attend college in another state. As Bill and Brandy's financial planner, which of the following funding programs would you advise them to avoid?

prepaid tuition plan. In all likelihood, Bill and Brandy should avoid a prepaid tuition plan because this type of plan typically requires that the child attend a public college or university within the state that established the plan. The other choices do not involve this type of restriction.

Max and Marlene have three children, Annie, Stephanie, and Joshua, attending college full-time. Which of the following statements regarding education tax credits is CORRECT? 1 An American Opportunity Tax Credit may be claimed by Max and Marlene for each child. 2 Max and Marlene may claim a Lifetime Learning Credit for the entire family.

Both In addition, a Lifetime Learning Credit may be claimed for one child and an American Opportunity Tax Credit may be claimed for the other two.

Melissa has $8,000 in a savings account, which she has earmarked for an Australian vacation later this year. The roof of her house was damaged recently and it requires considerable repairs. Melissa does not want to spend the money in her savings account to make the repairs because she believes that money is for her upcoming vacation. Instead, she withdraws $4,000 from her traditional IRA to make the repairs. She has to pay income tax of $1,120 plus a penalty of $400 on the IRA withdrawal. This is an example of which of the following behaviors?

Mental accounting Melissa's irrational financial decision resulted from mentally putting her money into separate "accounts" based on the function of those accounts. Self-control bias occurs when an individual lacks self-discipline and favors immediate gratification over long-term goals. Herding occurs when a person follows the actions of a larger group, whether rational or not. Confirmation bias means people tend to pay attention to information that supports their preconceived opinions while disregarding accurate, unsupportive information.

Maria would like to purchase a $225,000 home 10 years from today. To meet this goal, approximately how much should Maria invest at the end of each six-month period if she expects to earn an 8% annual rate of return, compounded semiannually, on her investments?

$7,556. This is an ordinary annuity calculation solving for PMT. The keystrokes on the HP 10bII/HP 10bII+ are as follows: END Mode 2, DOWNSHIFT, P/YR DOWNSHIFT, C ALL 225000, FV 8, I/YR 10, DOWNSHIFT, N = 20 Solve for PMT = −7,555.8938, or $7,556.

Your clients, Marshall and Rachel, file married filing jointly, and have three children, Allie, Sophie, and Jeffrey, attending college full-time. Which of the following statements regarding education tax credits is CORRECT? 1 Marshall and Rachel may claim a Lifetime Learning Credit for each child. 2 An American Opportunity Tax Credit may be claimed by Marshall and Rachel for each child.

2 The Lifetime Learning Credit is limited to one credit per taxpayer return. In addition, a Lifetime Learning Credit may be claimed for one child and an American Opportunity Tax Credit may be claimed for another.

Maxine, age 57, would like to retire in 10 years. Currently, her debt is decreasing as her cash flow and net worth are steadily increasing. Based on Maxine's current financial life cycle phase, which of the following goals is she likely to have?

Long-term goals, such as investing for retirement Maxine is in the conservation/protection phase of the financial life cycle. As such, her goals are likely longer-term goals, such as investing to provide for future retirement income. In the accumulation phase of the financial life cycle, clients have only limited discretionary income and, as a result, they are likely to focus on short-term, cost-of-living goals. Finally, in the distribution/gifting phase, estate planning and capital preservation are usually most important.

Wholesome Bakery supplies restaurants with bread and biscuits. The price of bread increases, resulting in a higher profit margin. As a result, the bakery chooses to reduce the supply of biscuits and to increase the supply of bread to take advantage of increased profits. Based on this information, which of the following statements is CORRECT?

The supply curve for biscuits would shift to the left. Other non-price related factors that may shift supply include technological advances, suppliers leaving the industry, and changes in the cost of the resources used to produce a good.

Blanca, a CFP® professional, has recently entered into a financial planning engagement with Simon. As his financial planner, which of the following are Blanca's roles? 1 Analyzing Simon's current financial status 2 Assisting Simon in implementing the financial plan 3 Helping Simon identify financial goals and objectives

all The financial planner is responsible for analyzing clients' financial status, assisting clients in implementing their financial plans, and making recommendations based on the client's goals and objectives.

In calculating the expected family contribution (EFC) for federal financial aid, which of the following parental assets is excluded? Life insurance policy cash values Retirement plan account balances

both Almost all parental assets are included in calculating the EFC, but these types of assets, along with home equity, are excluded.

Bill and Sophia have recently retired and want to travel to Europe and then volunteer for local mission work. They would also like to meet with Humphrey, their financial planner, to discuss charitable contributions they would like to make to these local missions. Humphrey should determine Bill and Sophia's life cycle phase to be

the distribution phase. The distribution/gifting phase begins subtly when a couple realizes that they can afford to spend on things they never believed possible. The asset accumulation and conservation/protection phases make this phase possible. For many people, there is a period when they are being influenced by all three phases simultaneously, though not necessarily to the same degree.

(Martin Case Study) Ryan is considering purchasing a variable universal life policy. Identify the licenses you must maintain to sell him the policy. 1 Series 24 2 Series 65 3 Series 6 or 7 4 A variable state insurance license

3,4 Generally, you must maintain a Series 6 or a Series 7 license and a variable state insurance license to sell variable universal life insurance. A Series 24 license is maintained by any person actively engaged in managing a member's securities or investment banking business, including supervising, soliciting, and conducting business, or in training persons associated with the member. A Series 65 license allows the holder to provide investment advice to clients within the holder's state of residence

Dionne, Danielle, and Daphne are currently college students eligible for federal student aid. Dionne owes a refund on a federal student grant. Danielle has been unable to maintain satisfactory academic progress. Daphne is experiencing financial difficulties and has defaulted on her federal student loan. Which of these students are likely to lose federal student aid eligibility?

all

Mandy, your financial planning client, would like you to advise her of tax-efficient estate planning strategies that preserve her family's wealth and incorporate charitable donations. What are the relevant elements in Mandy's financial planning engagement? 1 Address estate and legacy matters 2 Pursue philanthropic interests 3 Preserve or increase wealth 4 Identify tax considerations

all

Which of the following statements regarding interpersonal communication between financial planners and their clients are CORRECT? 1 Mirroring is accomplished by imitating the client's body language or verbal style. 2Body language can impact how clients receive and interpret messages more than any other type of communication. 3 Emotional intelligence includes the ability to recognize emotional expressions and select socially appropriate responses.

all Also, effective interpersonal communication involves the application of both oral and non-verbal skills, such as the effective use of body language.

In 2012, the average cost of a new home in Oakville was $150,000. In 2014 the average cost rose to $210,000. Due to an economic downturn in 2017, the average cost of a new home fell to $185,000, and the reaction to the decreased cost was positive, even though the new average cost was higher than the 2012 average cost of a new home. This behavior is known as

anchoring. When the average cost of a new home rose in 2014 to $210,000, home buyers reset their psychological anchors to that cost. As the price declined in 2017 to $185,000, the reaction was positive because it was considered in light of the higher 2014 price.

Derrick wants to establish an education fund that may be used to pay for his son's primary and secondary education expenses. Which of the following will meet Derrick's needs? 1 Section 529 plan 2 Coverdell Education Savings Account(CESA)

both Coverdell Education Savings Accounts (CESAs) and Section 529 plans may be used to pay for primary and secondary education expenses.

All of the following statements regarding the National Credit Union Share Insurance Fund (NCUSIF) are correct except A) The NCUSIF is backed by the full faith and credit of the U.S. government. B) The NCUSIF insures member accounts of all federal credit unions. C) The fund is administered by the National Credit Union Administration (NCUA). D) Up to $500,000 of a member's account balances are insured by the NCUSIF.

D The NCUSIF insures member accounts of all federal and most state-chartered credit unions up to $250,000, not $500,000.

Bryan wants to open a photography studio in four years. To do so, he needs to accumulate $175,000 (in today's dollars) to adequately finance this venture. He assumes he can earn an 8% compound annual after-tax rate of return on investment and inflation will average 3.5%. What will be Bryan's serial payment at the end of the third year?

$45,455.08. END mode FV= 175,000i= 4.3478 [(1.08 ÷ 1.035) − 1] × 100n= 4PV= 0PMTOA= (40,997.8820) × 1.035 = 42,432.8078 (change sign) 42,432.8078 × 1.035 = 43,917.9561 43,917.9561 × 1.035 = $45,455.08

Today, Ed put all of his cash into an account earning an annual interest rate of 4%, compounded monthly. Assuming he makes no withdrawals from or additions to this account, in approximately how many years will Ed double his money?

17.36. Ed will double his money in 17.36 years, with keystrokes on the HP 10bII/HP 10bII+ as follows: END Mode 12, DOWNSHIFT, P/YR DOWNSHIFT, C ALL 1, +/−, PV 2, FV 4, I/YR Solve for N = 208.29 months ÷ 12 = 17.36 years

You've determined the net present value (NPV) of your client's investment to be $32,500. If your client's required rate of return is 8%, which of the following is most likely to be the investment's internal rate of return (IRR)?

9%. If the net present value (NPV) of the investment is a positive number, the investment's IRR must be greater than or equal to the investor's required rate of return. Here, the investor's required rate of return is 8%, so the IRR must be greater than or equal to 8%.

To enact monetary policy, the Federal Reserve may use which of the following tools? 1 Taxation 2 Discount rate 3 Reserve requirements 4 Open market operations

2,3,4 Taxation is a tool used in fiscal policy decisions, which are made by Congress and the president (the Administration). All of the other tools are used by the Fed to enact monetary policy.

All of the following statements regarding interpersonal communication between financial planners and their clients are correct except A) mirroring is accomplished by imitating the client's body language or verbal style. B) emotional intelligence includes the ability to recognize emotional expressions and select socially appropriate responses. C) effective interpersonal communication involves the application of oral skills; it does not involve non-verbal skills. D) body language can impact how clients receive and interpret messages more than any other type of communication.

c it does not involve non-verbal skills. This statement is incorrect, because effective interpersonal communication involves both oral and non-verbal skills, such as the effective use of body language.

Christopher is presented with two equal investment opportunities. The first is stated in terms of potential gains, and the second is stated in terms of potential losses. Without having any additional information, Christopher selects the first investment. His decision reflects

loss aversion theory. The client's decision reflects the loss aversion theory, which states that people make decisions based on perceived gains rather than perceived losses. Herding occurs when a person follows the actions of a larger group, whether rational or not. Anchoring is making irrational decisions based on information that should have no influence on the decision at hand. The framing bias states that people are given a frame of reference, a set of beliefs or values, which they use to interpret facts or conditions as they make decisions.

*** Meghan is planning for her son's college education. She would like her son, who was born today, to attend a public university for four years beginning at age 18. Tuition is currently $15,000 a year and has increased at an annual rate of 6%, while inflation has only increased at 2.5% per year. Meghan can earn an investment rate of return of 9%. How much must she save at the end of each year if she wants to make the last payment at the beginning of her son's first year of college? Round to the nearest dollar.

$3,979. Keystrokes on the HP 10bII/HP 10bII+ are as follows: Uneven Cash Flow Method Step 1: Determine the PV of the years of tuition at age 18: 0, CFj 0, CFj 17, DOWNSHIFT, Nj 15000, CFj 4, DOWNSHIFT, Nj [(1.09 ÷ 1.06) − 1] × 100 = 2.8302 I/YR NPV = $34,834.5585 Step 2: Determine the annual payments needed to fund college tuition costs: 34834.5585, PV 18, N 9, I/YR PMTOA = ($3,978.5347), rounded to $3,979

*** John would like to purchase his friend's action figure collection in eight years for $12,000. His father will give him $2,000 toward the purchase of the collection at that time. John wants to save monthly for the balance of the purchase price. What is the approximate required monthly payment at the beginning of each month to accumulate the $12,000, including his father's gift, at an assumed interest rate of 5%?

$84.58. This requires an annuity due calculation. The amount to be funded is $10,000 ($12,000 - $2,000). The keystrokes on the HP 10bII/HP 10bII+ are as follows: BEG mode 12, DOWNSHIFT, P/YR DOWNSHIFT, C ALL 10000, FV 5, I/YR 8, DOWNSHIFT, N = 96 Solve for PMT = −84.5801, or $84.58.

In his financial planning practice, Frank allows his clients' goals and values to drive his relationships with them. He sees himself as a consultant. Frank's approach to financial counseling is known as

the strategic management approach. In this approach, the client's goals and values drive the client-planner relationship and the planner serves as a consultant. In the classical economics approach, planners attempt to achieve better financial outcomes by increasing financial resources or reducing expenditures. The cognitive-behavioral approach believes a client's attitudes, beliefs, and values influence their behavior and tries to replace negative beliefs with positive attitudes that should result in better financial results.

At the beginning of each year for the past 15 years, Betty, who is Angela and Avery's aunt, has put $5,000 into an account earning 5.5% annually for their benefit. How much is the account worth today and which process must take place to calculate the value?

$118,205.70; annuitizing The correct answer is $118,205.70; annuitizing. This is a future value of an annuity due calculation. The keystrokes on the HP 10bII/HP 10bII+ are as follows: BEG mode (DOWNSHIFT, Beg/End) 1, DOWNSHIFT, P/YR DOWNSHIFT, C ALL 5000, +/-, PMT 5.5, I/YR 15, DOWNSHIFT, N Solve for FV = 118,205.6999, or $118,205.70. This is done using annuitizing. Compounding is the process of interest being earned on increasing sums over time.

Which of the following CFP® certificants would likely be considered to be engaged in financial planning or the material elements of financial planning? 1 Lance, who reviews a life insurance sales brochure with his client, Arthur, and completes a variable life insurance application 2 Nadia, who conducts comprehensive data gathering regarding Jason's investments, life insurance, retirement plans, wills, and trusts and makes recommendations for him

2 Because Nadia's services involve several of the financial planning subject areas and she is involved in the elements of financial planning, she is likely providing financial planning. Lance's service to Arthur is limited, and the engagement would likely not be considered financial planning.

Mark is meeting with a new client, Macy. It is important that Mark understand Macy's psychological ability to deal with uncertain outcomes includes risk tolerance, risk capacity, and risk perception. During what step in the financial planning process should Mark measure Macy's abilities?

Understanding Macy's personal and financial circumstances

Mary has received a $75,000 inheritance from her grandfather, and she wishes to purchase a variable life insurance policy with Gilbert as the insured. She also would like to purchase a variable annuity and invest $25,000 of her inheritance in a mutual fund. Assume you have acquired the Series 6 and Series 63 licenses and are variable insurance licensed in the state. Which of the following are you allowed to sell to Gilbert and Mary? Mutual funds Variable annuities Variable life insurance policies

all Holding the FINRA Series 6 and Series 63 registrations and the appropriate variable insurance license qualifies an individual to sell mutual funds, variable life insurance policies, variable annuities, and initially offered unit investment trusts (UITs).

Doug and Cheryl have three dependent children studying at United University. Liam is a senior, Crissy is a junior, and Todd is a freshman. Doug and Cheryl are filing their taxes and want to use the American Opportunity Tax Credit and Lifetime Learning Credit to their greatest advantage. Assuming they qualify for both credits, which of the following statements is CORRECT?

they can use two American Opportunity Tax Credits for the qualified education expenses of Todd and Crissy and use one Lifetime Learning Credit for Liam. The American Opportunity Tax Credit can be used for qualified education expenses for only the first four years of post-secondary education for each student (a per student credit). The Lifetime Learning Credit is allowed once per year per family (a per tax return credit). However, the same expenses may not be used to qualify for both credits. Therefore, an American Opportunity Credit is available for two of the children and, in order to also use the Lifetime Learning Credit in the same year, the college expenses for the third child qualify the family for the that credit.

Which of the following assets would be inappropriate to include in an emergency fund for a family with $350 per month in discretionary income? 1 An S&P 500 Index mutual fund 2 A money market deposit account 3 Cash advance limit on consumer lines of credit 4 A certificate of deposit with a 2-year maturity

1,3,4 The only liquid and time-appropriate asset listed is a money market deposit account. An S&P 500 Index mutual fund should not be used for emergency fund purposes because its value may decline and any sale of shares may result in a taxable event. A certificate of deposit lacks liquidity and a cash advance on a consumer line of credit can be costly. LO 3.2.2

Which of the following are applicable FINRA examinations for registered investment advisers (RIAs)? Series 7: General Securities Registered Representative Series 66: Uniform Combined State Law Examination Series 63: Uniform Securities Agent State Law Exam Series 65: Uniform Registered Investment Adviser Examination

all The Series 65 is the Uniform Investment Adviser Exam. The registered investment advisor may also want to sell securities in which case the Series 7 and Series 63 would also be required for regulatory compliance and registration with states which the RIA was residing and transacting business. The Series 66 is the combination of the Series 63 and Series 65 combined into one exam.

Your client, Michael, would like to pay off his debt, reducing the debt with the highest interest rate first. Compared to the snowball approach of debt reduction, which of the following statements are CORRECT? This approach decreases the total amount of interest paid during the debt reduction process. Compared to the snowball approach, it is relatively more difficult to pay off the first debt with a high balance quickly.

both With Michael's approach, it often takes longer to pay off the first debt when the highest interest rate has a considerable balance. Less interest paid during the debt reduction process is an advantage of paying off debt in order of interest rate.

(Martin Case Study) Ryan and Nicole have decided to contribute equally to the cost of a $16,000 used car for Joshua when he graduates from college in two years. Ryan can earn a 5% annual after-tax rate of return. Calculate the amount Ryan must save at the beginning of each month, starting today, to reach this goal.

$316.32. Use BEG mode because this is an annuity due calculation. 12, DOWNSHIFT, P/YR DOWNSHIFT, C ALL 8000, FV (Ryan's share of the cost) 5, I/YR 2, DOWNSHIFT, N = 24 Solve for PMTAD = -316.32, or $316.32

Justin and Molly have become parents today with their first-born child, Harper. They would like to start saving for Harper's college education and would like her to attend college at National University for four years starting at age 18. As Justin and Molly's financial planner, you have determined that the total amount needed to fund Harper's college education is $175,000. If the expected annual return for deposits into a 529 plan is 9%, what is the amount of the quarterly deposits that must be made into a 529 plan?

$993.52. The payment is calculated as follows: END Mode 4, DOWNSHIFT, P/YR DOWNSHIFT, C ALL 175000, FV 9, I/YR 18, DOWNSHIFT, N = 72 Solve for PMT = -993.5239, or $993.52

Which of the following statements regarding behavioral finance is CORRECT? 1 It relates behavioral and cognitive psychology to financial planning. 2 It asserts that individuals generally make rational decisions regarding their finances.

1 It is traditional financial theory asserts that individuals generally make rational decisions regarding their finances. Yet financial planners often encounter cases where emotions and psychology have caused clients to make irrational choices when managing their money. Behavioral finance is a relatively new field of study which relates behavioral and cognitive psychology to financial planning and economics in an attempt to understand why people often act irrationally during the financial decision-making process.

Which of the following statements regarding the Expected Family Contribution (EFC) as it relates to student financial aid is CORRECT? 1 Parental assets include almost everything owned by the parents with notable exceptions. 2 Student income includes taxable and nontaxable income from the year preceding the award year, reduced by an income protection allowance and taxes. 3 Student assets include the value of everything the student owns or that has been saved on his behalf. 4 Parental assets and income are assigned a higher rating in the EFC calculation than student assets and income.

1,2,3 Parental assets and income are assigned a lower rating in the EFC calculation than student assets and income.

Oscar and Kathryn have a net worth of $250,000 before each of the following transactions: Purchased $3,000 of appliances on credit Transferred $2,000 from their checking account to a personal savings account Paid off a credit card with a balance of $6,000 using a personal savings account What is the net worth of Oscar and Kathryn after these transactions?

250,000 None of these transactions will change Oscar and Kathryn's net worth. The checking and savings account transactions offset each other and, although the addition of the appliances will increase the clients' assets by $3,000, the use of credit will increase their liabilities by the same amount.

Bernie is meeting with his clients, Mike and Alyssa, to define their goals. Mike tells Bernie that one of his goals is purchasing a hunting camp in two years and Alyssa shakes her head. What is the best action for Bernie to take next?

Ask Alyssa if the camp is a mutually agreed-upon goal. Alyssa's body language (shaking her head) may express that she does not agree with this goal. Therefore, Bernie should clarify whether or not she is on board with Mike's idea. If Alyssa is agreeable, Bernie should then get more details regarding the purchase of the camp. Bernie should not move on to other goals before Mike and Alyssa are in agreement regarding this particular one. Lastly, this is not the time to make recommendations without comprehensive information.

Craig is refinancing his current 7.5% 30-year fixed-rate mortgage for $150,000 into a new 5.75% 30-year mortgage for $150,000. How much is his new monthly payment and how much per month will he save on this payment?

New payment = $875.36, savings = $173.46 The correct answer is new payment = $875.36, savings = $173.46. END Mode 12, DOWNSHIFT, P/YR DOWNSHIFT, C ALL Old payment: 150000, PV 30, DOWNSHIFT, N = 360 7.5, I/YR Solve for PMT = −1,048.82 or $1,048.82 END Mode 12, DOWNSHIFT, P/YR DOWNSHIFT, C ALL New payment: 150000,PV 30, DOWNSHIFT, N = 360 5.75, I/YR Solve for PMT = −875.36 or $875.36 Savings = $1,048.82 − $875.36 = $173.46

Blake and Sarah have a monthly mortgage payments of $850 (principal, interest, taxes, and insurance [PITI]) on a mortgage balance of $95,000 on their home. They have an auto loan balance of $5,000, with monthly payments of $250. Additionally, they have a credit card balance of $2,000, on which they pay $225 each month. Blake and Sarah's net income for the past year was $35,000. Their gross income was $48,000. Are Blake and Sarah using excessive amounts of debt?

No, because monthly house payments (PITI) are less than 28% of gross income and total monthly debt payments are only 33% of gross income. Blake and Sarah are not using excessive amounts of debt. Both ratios should be calculated using gross income.

(Martin Case Study) From your meetings with Ryan, you realize he has a tendency to follow the actions of a larger group of people when making financial decisions, whether those actions are rational or not. Choose the behavioral finance theory that explains Ryan's behavior.

herding. Ryan's behavior is known as herding. Confirmation bias is the tendency to pay attention to information that supports one's preconceived opinions, while disregarding accurate, unsupportive information. Overconfidence occurs when an investor considers his abilities to be much better than they actually are. Anchoring occurs when a person makes an irrational decision based on information that should have no influence on the decision.

(Martin Case Study) Assume the Martins financed the purchase of an $1,300 refrigerator using a new line of credit paid $1,000 of their credit card debt using a gift from Emile and Dee bought an antique vase, valued at $400, for $200 using their checking account debit card Calculate the Martins' net worth after these transactions.

$217,753. Paying $1,000 of credit card debt reduces the Martins' liabilities by this amount. This has a positive effect of $1,000 on net worth. Although the purchase of the refrigerator will increase the Martins' assets by $1,300, the use of credit will increase liabilities by the same amount; therefore, this transaction has no effect on net worth. Using checking account funds (via a debit card) of $200 to purchase the antique vase valued at $400 will result in an increase in assets of $200 ($400 - $200). This results in a total net worth of $217,753 ($216,553 net worth from Statement of Financial Position + $1,000 + $200).

*** Annie deposits $2,500 into an account earning an annual interest rate of 6.25% compounded monthly. Assuming she makes no withdrawals or additions to this account, approximately how many years will it take for Annie to have $5,000 in her account?

11.12 years. This is a number of periods calculation. The keystrokes on the HP 10bII/HP 10bII+ are as follows: END Mode 12, DOWNSHIFT, P/YR DOWNSHIFT, C ALL 2500, +/-, PV 5000, FV 6.25, I/YR Solve for N = 133.4305 months ÷ 12 = 11.1192, or 11.12 years.

Which of the following is an example of nondiscretionary expenses? 1 Concert tickets 2 Utility bills 3 Charitable contributions 4 Auto loan payments

2,4 Utilities, and auto loan payments are all considered nondiscretionary expenses. Concert tickets and charitable contributions are generally not necessary expenses; therefore, they are considered discretionary.

*** Which of the following financial statements provides a snapshot of the client's net worth at any given point in time, usually at the end of a calendar year? 1 Personal tax return 2 Cash flow statement 3 Net worth statement 4 Statement of financial position

3-4 A statement of financial position, also known as a personal balance sheet or net worth statement, provides a snapshot of the client's net worth at any given point in time, most often at the end of a calendar year.

Which of these are examples of discretionary expenses? Taxes Utility bills Health club dues Mortgage payments

Health club dues Health club dues are generally not a necessary expense; therefore, they are considered discretionary. Taxes, utilities, and mortgage payments are all considered nondiscretionary expenses.

Which of the following expenses qualify for an employer's Educational Assistance Program? 1 Enrollment fees 2 On-campus housing 3 Full-time, graduate school tuition 4 Part-time, undergraduate tuition

1,3,4 With the Educational Assistance Program, an employer can reimburse an employee's undergraduate and graduate tuition, enrollment fees, books, supplies, and equipment, and these benefits are excluded from the employee's income up to $5,250 per year. Meals, transportation, and lodging are not qualified.

(Martin Case Study) Assume the Martins paid $2,000 of their credit card debt using a gift from Emile and Dee; financed the purchase of an $800 laptop using a new line of credit; and bought Justin a vintage trombone, valued at $500, online for $300 using their checking account debit card. Calculate the Martins' net worth after these transactions.

$218,753. Paying $2,000 of credit card debt reduces the Martins' liabilities by this amount. This has a positive effect of $2,000 on net worth. Although the purchase of the laptop will increase the Martins' assets by $800, the use of credit will increase liabilities by the same amount; therefore, this transaction has no effect on net worth. Using checking account funds (via a debit card) of $300 to purchase the vintage trombone valued at $500 will result in an increase in assets of $200 ($500 − $300). This results in a total net worth of $218,753 ($216,553 net worth from Statement of Financial Position + $2,000 + $200).

Which of the following activities would be appropriate if you were establishing and defining the client-planner relationship or gathering information necessary to fulfill the engagement? 1 Collecting personal financial information 2 Inquiring about the number of dependents 3 Inquiring about the age or dates of birth of dependents 4 Determining which stocks to purchase for the client's investment portfolio

1-3 Establishing and defining the client-planner relationship does not include determining which stocks or investments to purchase. This occurs in the fourth phase of the financial planning process, developing the recommendations.

Stacey has a team of financial advisors which consists of a financial planner, an estate planning attorney, and a CPA. In this case, the financial planner's responsibilities for Stacey include which of the following? 1 Drafting a power of attorney for Stacey 2 Analyzing Stacey's current financial status 3 Assisting Stacey as she identifies her financial goals 4 Monitoring whether Stacey is following her plan after it is implemented

2-4 There is no mention that Stacey's financial planner is a licensed attorney. Unless this is the case, the planner should not draft legal documents such as powers of attorney and wills to avoid the unauthorized practice of law.

*** (Martin Case Study) The Martins ask you whether their consumer debt ratio is at an acceptable level without considering the projected income earned by Sherri. What should you advise them? (Exclude any Section 401[k] plan loan payments.)

8.53%, which is excellent. Monthly debt payments Debt payments* = $80 + $253.33 = $333.33 Monthly net income ($71,050 - [$22,855 + $1,320]) / 12 = $3,906.25 Consumer debt ratio = $333.33 / $3,906.25 = 8.53% NOTE: Monthly credit card payments, boat loan payments, and all sources of income from Statement of Cash Flows without considering the projected income earned by Sherri. A generally accepted rule in personal financial planning is that the consumer debt ratio should not exceed 20%. Thus, a consumer debt ratio of 8.53% is considered excellent.

*** Peter, age 35, has requested your expertise in developing a college funding plan for his five-year-old daughter, Brooke. He has presented you with the following information. Current annual salary—$115,000 Monthly mortgage payment—$1,700 Credit card debt—$3,000 (16.5% fixed) Checking account balance—$1,345 Long-term group disability insurance—60% of salary to age 65, 60-day elimination period Life insurance—1x salary (group), $400,000 20-year term (individual) After completing a budget with Peter, you have determined that he has $350 per month in surplus cash flow. He tells you he would like to use this amount to fund a college plan for Brooke. Based on the information provided, what should Peter do first?

Establish an emergency fund At this point, Peter should use his surplus cash flow to establish an emergency fund. He does not have access to immediate cash, and, in the event of disability, he does not have enough set aside to cover his elimination period. Paying off his credit card debt, purchasing additional life insurance, and establishing a college plan for Brooke should be considered after establishing the emergency fund.

Janice, a CFP® professional, offers comprehensive financial planning services to her clients in her Registered Independent Advisory firm. Which of the following services that Janice provides are considered relevant elements of financial planning according to the CFP Board Code and Standards? Developing client goals Preserving or increasing wealth Providing for educational needs Identifying and managing the financial effect of health considerations

all

Input from a team of financial advisors may be needed during the development of a comprehensive financial plan. This team of financial advisors, employed by the client, may include which of the following professionals? A life insurance agent A private money manager A Certified Public Accountant An estate planning attorney

all This team may also include other financial professionals, such as a property and casualty insurance agent or a banker.

Wes is considering investing in a new printing press for his printing business. The purchase price of the printing press is $225,000 and he expects to be able to sell it for $150,000 at the end of five years. During the five-year period, he expects the equipment to increase her annual cash flows by $45,000 (year 1), $32,000 (year 2), $24,000 (year 3), $16,000 (year 4), and $10,000 (year 5). If his opportunity cost is 7%, what is the net present value (NPV) of this investment?

-$9,118.62. The keystrokes on the HP 10bII/HP 10bII+ are as follows: 225,000 +/- CFj 45,000 CFj 32,000 CFj 24,000 CFj 16,000 CFj 160,000 CFj (year 5; final annual cash flow of $10,000 plus the anticipated sales price of $150,000); 7 I/YR [DOWNSHIFT] NPV = -$9,118.6248, or -$9,118.62.

Which of the following statements regarding a financial planner's analysis of a client's cash flow statement is CORRECT? 1 The analysis of the client's cash flow statement can help the planner determine whether the client is living within his financial means. 2 The analysis of the client's cash flow statement helps determine the client's net worth, or total cash surplus, by tracking cash inflows and outflows over a period of time. 3 Typically, the financial planner will encourage the client to reduce the variable expenses reported on the cash flow statement

1, 3 The analysis of the client's cash flow statement helps determine the client's savings level, or total cash surplus, by tracking cash inflows and outflows over a period of time. Net worth is determined in a statement of financial position.

*** Candice has referred Rochelle, a CFP® professional, to her brother, Nelson. In their initial meeting, Rochelle explains how she can help Nelson develop a comprehensive financial plan. Which of the following would be Rochelle's roles in a client-planner relationship with Nelson? 1 Assisting Nelson in identifying his goals 2 Analyzing Nelson's current financial status 3 Recommending strategies that will meet Nelson's business goals 4 Providing documentation Rochelle needs to complete the financial plan

1,2,3 Rochelle is responsible for helping Nelson with identifying his goals and making recommendations based on those goals. She is also responsible for analyzing Nelson's current financial status. Nelson has the duty to provide documentation Rochelle needs to complete the financial plan.

Which of the following statements regarding people who have a visual learning style is CORRECT? 1 They tend to respond to graphs, charts, pictures, and reading information. 2 They retain information by hearing or speaking. 3 They express themselves through facial expressions. 4 They prefer their goals and objectives to be presented as a to-do-list in bullet form.

1,3 Statement II is incorrect because people who retain information by hearing or speaking have an auditory learning style. Statement IV is not correct because individuals who prefer goals and objectives to be presented in bullet form exhibit a kinesthetic learning style.

Brandon's employer is willing to assist him in the cost of higher education. What is the maximum that Brandon's employer can provide to him in educational assistance and qualify for the employee exclusion from income?

$5,250. An employer may provide up to $5,250 of educational assistance to an employee during any one year as a tax-free employee benefit under Section 127 of the Internal Revenue Code. This benefit may be provided either for job-related or non-job-related education and may also be granted for either undergraduate or graduate study.

Kim wishes to accumulate a lump-sum amount of $50,000 (in today's dollars) in five years for a down payment on a vacation villa. She assumes an inflation rate of 2.5% and a before-tax investment rate of return of 6%. If she deposits the needed savings at the end of each year for the next five years, what is her required payment in the first year using the serial payment method?

$9,573.49. The required annual deposit under the serial payment approach is $9,573.49, with keystrokes as follows on the HP 10bII/HP 10bII+: END Mode 1, DOWNSHIFT, P/YR DOWNSHIFT, C ALL 50000, FV [(1.06 ÷ 1.025) − 1] × 100 = 3.4146, I/YR 5, DOWNSHIFT, N Solve for PMT = −9,399.99; PMT of −9,399.99 × 1.025 = −9,573.49, or $9,573.49.

Brittany, Chelsie's daughter, will be attending college in the next few years. Chelsie would like to know more from Brandon, her financial planner, about education grants. Which of the following are accurate statements regarding education grants that Brandon can make to Chelsie? 1 Pell Grants are available to undergraduate students only. 2 Federal Work-Study Programs provide students with part-time jobs while attending college. 3 Pell Grants are disbursed to the educational institution for the benefit of the qualifying student. 4 Federal Supplemental Educational Opportunity Grants (FSEOGs) are federal grants for which priority is given to students who also receive Pell Grants.

1,2,4 Pell Grants are made directly to students and are available to undergrads only. FSEOGs, which are federal grants, give priority to students who also receive Pell Grants. Federal Work-Study Programs provide students with part-time jobs while attending college. In turn, the institution disburses the earned funds to the students.

Creed wants his debts canceled entirely through bankruptcy. Under this form of bankruptcy, which of the following assets are exempt from creditor claims? 1 Disability income benefits 2 Student and government loans 3 Proceeds from any annuity contracts 4 Federal civil service retirement benefits

1,3,4 This is a Chapter 7 bankruptcy. Student and government loans are not generally dischargeable in this type of bankruptcy. All of the other assets are generally exempt from creditor claims in bankruptcy. A person's homestead, pension, and retirement plan rights (ERISA plans) and some limited amount of personal property are also exempt.

Which of the following statements regarding financial institutions is CORRECT? 1 A brokerage company is an intermediary that facilitates transactions involving sales of investments or real estate. 2 A credit union, owned by its members, is a financial institution that accepts deposits and makes loans 3 Federally chartered S&Ls are regulated by the Office of the Comptroller of the Currency. 4 A trust company is also known as a thrift institution.

1-3 A savings and loan association is also known as a thrift institution. This is not true of trust companies.

Dan and Dana are preparing their statement of financial position. Currently, they have short-term assets of $15,000, long-term assets of $150,000, short-term debt of $12,000, and long-term debt of $125,000. Based on the information provided, what is Dan and Dana's current ratio and it is acceptable?

1.2; acceptable. The current (short-term) ratio is determined by dividing current assets by current (short-term) liabilities: $15,000 ÷ $12,000 = 1.25. The acceptable target for this ratio is between 1.0 and 2.0.

(Martin Case Study) Ryan asks you about a Coverdell Education Savings Account (CESA) for college savings. To avoid income taxation and penalties, you advise him that these funds must be used before students reach what age?

30. All funds within a CESA must be used before the student reaches age 30. Any remaining funds will be disbursed to the CESA beneficiary, and the earnings will be subject to income tax and a 10% penalty.

(Martin Case Study) Ryan has received a $50,000 inheritance from a brother, and he wishes to purchase a variable life insurance policy naming Joshua as the insured. He also would like to purchase a variable annuity and is considering investing in a unit investment trust (UIT). Assume you have acquired the Series 6 and Series 63 licenses and are variable insurance licensed in the state. Which of the following are you allowed to sell to Ryan? Mutual funds Variable annuities Initially offered unit investment trusts (UITs)

all Holding the FINRA Series 6 and Series 63 registrations and the appropriate variable insurance license qualifies an individual to sell mutual funds, variable life insurance policies, variable annuities, and initially offered unit investment trusts (UITs).

Bobbi is a full-time employee at an insurance agency and is enrolled in night classes on a part-time basis in pursuit of her MBA. Her employer offers reimbursements for qualified expenses through the Educational Assistance Program. This year Bobbi has incurred the following educational expenses: Books: $800 Tuition: $3,000 Enrollment Fees: $200 Computer Lab Fee: $100 Clothing for school: $150 Based on these expenses, identify the amount of reimbursement for which Bobbi can receive under the Educational Assistance Program.

$4,100. An employer can reimburse an employee's tuition (both graduate and undergraduate), enrollment fees, books, supplies, and equipment, and these benefits are excluded from the employee's income up to $5,250 per year. Bobbi's qualifying expenses equal $4,100 ($200 + $3,000 + $800 + $100).

John, age 55, is unmarried and retired. He has the following assets on deposit at Allworld Bank, an FDIC-insured financial institution: Checking account - John $80,000 Savings account - Joint with son $70,000 Certificate of deposit (CD) - John $225,000 Rollover traditional IRA - John $150,000 What amount is insured by the FDIC?

$470,000. The FDIC insures separate legal categories of accounts of a legal institution. As a result, the individual accounts owned by John (CD and checking account) are aggregated and are insured up to a total of $250,000. The joint account is insured for $70,000. The individual retirement account (IRA) will be insured up to $250,000. Total amount insured is $470,000 ($250,000 + $70,000 + $150,000).

Which of the following education funding plans provides tax advantages regardless of the contributor's adjusted gross income (AGI)? 1 Section 529 plan 2 Series EE or Series I savings bonds 3 Coverdell Education Savings Account (CESA)

1 This plan offers significant tax advantages regardless of the contributor's adjusted gross income (AGI). The tax advantages of CESAs and education savings bonds are phased out at higher levels of AGI.

The degree to which a client's financial resources can cushion risks is known as

risk capacity. Risk capacity is the degree to which a client's financial resources can cushion risks. A client's assessment of the magnitude of the risks being traded off is known as risk perception. Risk tolerance refers to the trade-offs clients are willing to make between potential risks and rewards. Emotional intelligence is the ability to recognize emotional expressions in oneself and the client and to select socially appropriate responses to both the circumstances and the client's emotions.

The behavioral finance concept that asserts people are given a frame of reference, a set of beliefs or values, which they use to interpret facts or conditions as they make decisions, is known as

framing bias. This asserts that people are given a frame of reference, a set of beliefs or values, which they use to interpret facts or conditions as they make decisions. Anchoring is making irrational decisions based on information that should have no influence on the decision at hand. Confirmation bias is paying attention to information that supports a preconceived opinion and poorly made decision, while disregarding accurate, unsupportive information. Mental accounting is putting money into separate "accounts" based on the function of these accounts.

The Federal Reserve Bank exercises its monetary policy by directly controlling which of the following interest rates? 1 Prime rate 2 Discount rate 3 Federal funds rate

2 The Federal Reserve Board can adjust the discount rate (the rate one Federal Reserve Bank charges to others) in its efforts to control the money supply. The Fed does not directly control the federal funds rate (the rate charged between banks on short- term loans). The Fed does not control the prime rate, which is the rate commercial banks charge their best customers.

Which of the following statements regarding the business cycle is CORRECT? 1 Usually, the economy is either in an expansion or contraction phase. 2 The business cycle reflects movements in economic activity and illustrates the concepts of supply and demand.

both

Claudio, a CFP® professional, met with Hector and Hildie for their first meeting to provide an overview of the comprehensive financial planning services available at his firm and get an idea of Hector and Hildie's financial circumstances. During their conversation, Hector and Hildie expressed an interest in investing in mutual funds. Claudio frequently advised Hector and Hildie that his practice is designed to offer customized financial planning. Despite the various descriptions of planning services offered by Claudio, the couple remained focused only investing and were unwilling to complete a client agreement to engage in financial planning relationship with Claudio. Analyze these facts and, according to the Code and Standards, determine Claudio's options for engaging Hector and Hildie as clients. 1 Claudio can accept Hector and Hildie as financial planning clients without the client agreement since investing is a component of his comprehensive planning services. 2 Claudio may provide the requested services after informing Hector and Hildie how financial planning would be beneficial and how the decision not to engage his services to provide financial planning may limit his financial advice. 3 Claudio can choose to not enter into an engagement. 4 Claudio can limit the scope of engagement to services that do not require application of the Practice Standards, and describe to Hector and Hildie the services that they request that he will not be performing.

2-4 The correct answer is II, III, and IV. According to Standard B.6 of the Code and Standards (No Client Agreement to Engage for Financial Planning): If a CFP® professional otherwise must comply with the Practice Standards, but the Client does not agree to engage the CFP® professional to provide Financial Planning, the CFP® professional must either: Not enter into the Engagement; Limit the Scope of Engagement to services that do not require application of the Practice Standards and describe to the Client the services the Client requests that the CFP® professional will not be performing; Provide the requested services after informing the Client how Financial Planning would benefit the Client and how the decision not to engage the CFP® professional to provide Financial Planning may limit the CFP® professional's Financial Advice, in which case the CFP® professional is not required to comply with the Practice Standards; or Terminate the Engagement. If Claudio elects to take Hector and Hildie as clients, under the Code and Standards, he is, at least, obligated to describe how his ability to perform Financial Planning will be limited or describe the services he will not be performing (after limiting the Scope of the Engagement). He cannot simply move forward without a client agreement in place.

Daniel secures a $350,000 30-year mortgage with an annual interest rate of 6%. How much total interest will Daniel have paid on the mortgage at the end of 25 years?

$388,070. Daniel will have paid $388,070.37 in total interest on the mortgage at the end of 25 years. The keystrokes on the HP 10bII/HP 10bII+ as follows: END mode 12, DOWNSHIFT, P/YR DOWNSHIFT, C ALL 350000, PV 30, DOWNSHIFT, N = 360 6, I/YR Solve for PMT = -2,098.4268 1, INPUT, 300 DOWNSHIFT, AMORT (pressing the = key toggles you through amortization totals for months 1 through 300); = -241,457.6682 (total principal paid through 300 months) = -388,070.3718 (total interest paid through 300 months) = 108,542.3318 (remaining principal balance through 300 months).

Eric, who turns two today, will begin college at age 18. Eric's parents, Martin and Ellie, want to begin a college education savings program for Eric. Currently, college costs are $30,000 per academic year. The couple assumes that college costs will increase at the rate of 3% annually from now until Eric enters college. They also assume they can achieve an after-tax rate of return of 7.5% annually on funds earmarked for this goal. They expect Eric will attend college for four years. What is the payment that must be made at the end of each year until he enters college at age 18?

$6,218. The keystrokes on the HP 10bII/HP 10bII+ calculator are as follows: Determine the future cost of college for the first year: END Mode 30000, +/−, PV 3, I/YR 18 − 2 = 16, N Solve for FV = 48,141.1932, rounded to $48,141.19 Determine the account balance necessary to fund college education: BEG Mode 48,141.19, +/−, PMT 4.3689, I/YR (Use the inflation-adjusted rate of return: [(1.075 ÷ 1.03) − 1] × 100 = 4.3689%). 4, N Solve for PV = 180,807.4644, rounded to $180,807.46 Determine the required savings payments: END mode 180,807.46, FV 16, N 7.5, I/YR Solve for PMT = 6,218.1778, rounded to $6,218

An investor makes an initial deposit of $20,000 into a mutual fund. Each subsequent year, he deposits an additional $2,500 into the fund. What will be the value of the account in eight years if the fund earns 9% annually?

$67,422.44. END Mode 1, DOWNSHIFT, P/YR DOWNSHIFT, C ALL 8, DOWNSHIFT, N 9, I/YR 20000, +/-, PV 2500, +/-, PMT Solve for FV = $67,422.44 The account value after eight years would be $67,422.44.

Matilda is considering investing in a piece of equipment for her business. The purchase price of the equipment is $100,000 and she expects to be able to sell it for $40,000 at the end of five years. During the five-year period, Matilda expects the equipment to increase her annual cash flows by $25,000, $30,000, $20,000, $15,000, and $10,000. If her opportunity cost is 8%, what is the net present value (NPV) of this investment?

$9,799.57. The NPV of this investment is $9,799.57. The keystrokes on the HP 10bII/HP 10bII+ are as follows: 100,000 +/- CFj (year 0); 25,000 CFj (year 1); 30,000 CFj (year 2); 20,000 CFj (year 3); 15,000 CFj (year 4); 50,000 CFj (year 5; final annual cash flow of $10,000 plus the anticipated sales price of $40,000); 8 I/YR [DOWNSHIFT] NPV = 9,799.5652, or $9,799.57.

Select the group(s) to which the CFP Board Fitness Standards apply. 1 Candidates for the CFP® exam 2 Professionals Eligible for Reinstatement 3 Respondents in a CFP Board investigation 4 A CFP® practitioner acting in a supervisory capacity

1,2 The Fitness Standards apply to candidates for the CFP® exam and Professionals Eligible for Reinstatement (PER). PER include both individuals who are not currently certified but have been certified by CFP Board in the past and individuals are eligible to reinstate their certification without being required to pass the current CFP® Certification Examination

Which of the following correctly states the relationship between NPV, IRR, and required return? 1 If NPV > 0, then IRR > required return. 2 If NPV = 0, then IRR = required return. 3 If NPV > 0, then IRR < required return. 4 If NPV < 0, then IRR < required return.

1,2,4 Only statement III is incorrect. If the NPV is a positive number, then the investor will earn a rate of return (IRR) greater than the required rate of return. Conversely, if the result of the NPV calculation is a negative number, then the investor will earn a rate of return (IRR) less than the required rate of return.

Which of the following statements regarding Statements of Cash Flows is CORRECT? 1 The cash flow statement helps to determine a client's savings level. 2 The three components of the cash flow statement are assets, liabilities, and net worth. 3 A statement of cash flows represents a "snapshot" of a client's status as of a given date. 4 "For the period January 1, 20XX to December 31, 20XX" is an appropriate way to indicate the period covered by the cash flow statement.

1,4 It is the statement of financial position which represents a "snapshot" of a client's status on a particular date. The three components of the cash flow statement are cash inflows, cash outflows, and net cash flow. The cash flow statement reflects the client's financial activity over a period of time. The correct way to indicate the period covered is: For the period January 1, 20XX to December 31, 20XX. Usually the cash flow statement is prepared for the calendar year period.

Kristoff has been advised by his financial planner that he needs to increase his savings. Which of the following are ways Kristoff can save for this purpose? 1 Cancel his video game subscription 2 Choose a more economical cable channel plan 3 Use his overdraft feature on his debit cards 4 Decrease the deductible on his homeowners policy

1-2 Using an overdraft feature on debit cards may tempt Kristoff to spend money he does not have available in his account. Decreasing insurance deductibles increases premiums, which is not a savings strategy.

Gregory is meeting with his client, Marlon. During a discussion of Marlon's estate planning, Marlon asks, "Would my sister be a good choice for trustee?" The planner answers, "What do you feel are her qualifications to serve as a trustee?" Gregory's answer is an example of

leading response. Gregory's answer is a leading response because it guides Marlon to provide more detail. Body language involves facial expressions, eye contact, gestures, and body posture. Verbal mirroring is imitating the client's word use, tone of voice, and communication method. Emotional intelligence is the ability to recognize emotional expressions in one's self and others and select socially appropriate responses.

Benton has just landed a stable job with growth potential. He and his spouse, Laura, would like to purchase a new home near Benton's office. They are meeting with their loan officer to determine which mortgage would best suit their needs. Laura is a stay-at-home mom raising their three school age children. She sometimes provides housekeeping services for her immediate family members. Benton and Laura want a mortgage that offers the lowest monthly payment with a fixed interest rate. Based on this information, which mortgage would be the most appropriate choice?

30-year conventional mortgage. Generally, the 30-year conventional mortgage will have a lower payment (principal + interest) compared to the 15-year conventional mortgage. A client who currently has stable cash flow and wants to have a predictable mortgage payment each month should use a conventional fixed-rate 30-year mortgage, for which the principal is completely paid off at the end of the term. The question does not indicate that either Benton or Laura is eligible for a VA (Veterans Administration) loan.

Brandon has been advised by his financial planner that he needs to increase his savings. Which of the following are ways Brandon can save for this purpose? Use his overdraft feature on his debit cards Decrease the deductible on his homeowners policy

neither Using an overdraft feature on debit cards may tempt Brandon to spend money he does not have available in his account. Decreasing insurance deductibles increases premiums, which is not a savings strategy.

Rafael is planning to finance his new home with a mortgage from ABC Bank. When meeting with his mortgage banker, he is given a loan packet detailing the amount to be financed, the annual percentage rate (APR), and the loan's terms and conditions. What legislation requires the bank to disclose this information to Rafael?

Consumer Credit Protection Act. Also known as the Truth in Lending Act, the Consumer Credit Protection Act requires lenders, before extending credit, to disclose both the dollar amount of finance charges and the annual percentage rate (APR), as well as other loan terms and conditions. The act also limits consumer liability for a lost or stolen credit card to the amount charged or a maximum of $50 per card, whichever is less. The requirements of this act are often encountered when a consumer enters into a mortgage agreement with a lender and closes on a personal residence.

Which of the following statements regarding The Securities Act of 1933 are CORRECT? 1 The Act provides applicable procedures for issuing an initial public offering (IPO) of securities. 2 The Act identifies securities that are exempt from the registration requirements. 3 The Securities and Exchange Commission (SEC) was established by this act. 4 The Act requires the registration of new issues of securities.

I, II, and IV. The SEC was established as the primary regulatory body overseeing the sale and purchase of securities by investors under The Securities Exchange Act of 1934.

Several financial planners attended a charitable gala last month. During the event, they introduced themselves to one another. Marla, an attorney, advertises that she also provides investment advice to clients. Brenda, who provides advice regarding securities issued and guaranteed by the U.S. government only. Arthur introduced himself as an attorney who solely advises his clients regarding venture capital opportunities. Lara is a Certified Public Accountant who primarily provides investment advice and security analysis. Misty is a CFP® professional who receives compensation for publishing reports upon request by investors. Who must register as an investment adviser with the SEC?

Marla, Lara, and Misty only. Because Marla advertises that she gives investment advice, this service is not incidental to her law practice. Brenda need not register because her advice is limited to securities issued and guaranteed by the U.S. government. This is an exception to registration. Misty's reports are specific to those who request them; therefore, she must register. As Lara is principally involved in offering investment advice and security analysis, she is required to register. Because Brenda's advisory services are solely incidental to her accounting practice, she is excluded from the definition of an investment advisor. Arthur solely advises his clients regarding venture capital opportunities; therefore, he is exempt from registering as an investment adviser.

Mitch and Candace, your clients, are ages 48 and 50, respectively. They want to invest $7,000 each year toward the future college expenses of their eight-year-old daughter, Rachel, who will begin college in 10 years. Mitch and Candace had hoped that Rachel would get a soccer scholarship because she is a talented athlete. Unfortunately, due to Rachel's recent ankle injury, her parents have decided to plan for the likely event that she will not receive an athletic scholarship. Mitch and Candace do not feel Rachel would qualify for financial aid. Which of the following investments would best meet their needs?

Roth IRA for Candace. It is the best choice because, for financial aid purposes, it keeps an asset out of Rachel's name. Also, in 2021, the Roth IRA allows contributions up to $6,000 with an additional catch-up amount of $1,000 for individuals at least 50 years old. Because Candace is age 50, she qualifies for the $1,000 catch-up and the full $7,000 may be deposited each year.

Select the Standard that the CFP Board has identified as the "cornerstone of the Code and Standards."

fiduciary duty. The cornerstone of the Code and Standards is the Fiduciary Duty. At all times when providing Financial Advice to a Client, a CFP® professional must act as a fiduciary and, therefore, act in the best interests of the Client.

To be eligible to receive federal student aid, you must have a valid Social Security Number. maintain satisfactory academic progress. be a citizen or eligible noncitizen of the United States. be enrolled in an eligible program as a regular student pursuing a certificate.

all To be eligible to receive federal student aid, you must be a citizen or eligible noncitizen of the United States; -have a valid Social Security Number; -have a high school diploma or a General Education Development (GED) certificate, or have completed homeschooling. If you don't, you may still be eligible for federal student aid if you were enrolled in college or career school prior to July 1, 2012. Go to https://studentaid.ed.gov/eligibility/basic-criteria for additional information; -be enrolled in an eligible program as a regular student seeking a degree or certificate; -maintain satisfactory academic progress; -not owe a refund on a federal student grant or be in default on a federal student loan; -register (or already be registered) with the Selective Service System, if you are a male and not currently on active duty in the U.S. Armed Forces; and -not have a conviction for the possession or sale of illegal drugs for an offense that occurred while you were receiving federal student aid (such as grants, work-study, or loans). If you have such a conviction, you must complete the Student Aid Eligibility Worksheet to determine if you are eligible for aid or partially eligible for aid.

Which of the following statements regarding an economic recession and a depression is CORRECT? A depression occurs when the GDP has experienced a decrease for 6 consecutive quarters or a minimum of 18 months from a baseline of zero. A recession occurs when the GDP has experienced a decrease for two consecutive quarters or a minimum of six months from a baseline of zero.

both

Connor and Caroline would like to invest $800 monthly into a 529 plan for their seven-year-old son, Hank. They expect Hank to attend American University, where the current tuition is $25,000 per year. Education inflation is expected to be 6.5% and the anticipated rate of return on their 529 plan is 8%. Hank will attend school for four years, starting at age 18. Based on these facts will Connor and Caroline's current investment plan with $800 monthly deposits meet their education savings goal?

no, it will fall short of the goal by $27,324.36. The calculations are as follows: Step 1: Determine the future cost of college for the first year: END Mode 1, DOWNSHIFT, P/YR DOWNSHIFT, C ALL 25000, +/-, PV 6.5, I/YR 11, N Solve for FV = 49,978.7850, or $49,978.79 Determine the account balance necessary to fund college education: BEG Mode (money is needed at the beginning of college) 49,978.7850, +/−, PMT [(1.08 ÷ 1.065) − 1] × 100 = 1.4085, I/YR 4, N Solve for PV = 195,788.6712, or $195,788.67 Step 2: Calculate the future value of the current payments: 800, +/−, PMT 8, I/YR (11 × 12) 132, N Solve for FV = 168,464.3135, or $168,464.31 Step 3: Subtract the future education need from future value of current payments. $168,464.31 − $195,788.67= −$27,324.36

Which of the following statements regarding the decision to buy or lease a home are CORRECT? 1 The itemized tax deduction for mortgage interest is a benefit of home ownership. 2 A client who will be residing in the home for a short period of time should lease the home. 3 The lower the marginal income tax bracket, the greater the advantage of owning a home. 4 Generally, more costs are associated with buying a home, especially if this arrangement is short term.

1,2,4 Individuals in higher marginal income tax brackets benefit more from owning a home than those in lower tax brackets. More costs are associated with purchasing a home, regardless of the length of the lease.

Reinstatement by CFP Board is possible under which of the following circumstances? 1 Revocation 2 Suspension of one year or less 3 Suspension of more than one year 4 Temporary bar of more than one year

2, 3, 4 Revocation is permanent. There will be no opportunity for reinstatement.

Phil secures a $350,000, 15-year mortgage with an annual interest rate of 5.5%. What will be the unpaid principal balance on Phil's mortgage at the end of 10 years?

$149,718.22. The unpaid principal balance on Phil's mortgage at the end of 10 years will be $104,761.78. The keystrokes on the HP 10bII/HP 10bII+ are as follows: END mode 12, DOWNSHIFT, P/YR DOWNSHIFT, C ALL 350000, PV 15, DOWNSHIFT, N = 180 5.5 , I/YR Solve for PMT = -2,859.7921 1, INPUT, 120 DOWNSHIFT, AMORT; (pressing the = key toggles you through amortization totals for months 1 through 120); = - 200,281.7762 (total principal paid through 120 months) = -142,893.2758 (total interest paid through 120 months) = 149,718.2238 (remaining principal balance through 120 months of payments)

Frank purchased an antique table today for $30,000. Experts advised him the table will increase in value at a rate of 3% annually for the next six years. Approximately how much will the table be worth at the end of the sixth year if his expected return is achieved?

$35,822 The correct answer is $35,822. This is a future value of a single sum calculation. The keystrokes on the HP 10bII/HP 10bII+ are as follows: END Mode 1, DOWNSHIFT, P/YR DOWNSHIFT, C ALL 30000, +/-, PV 3, I/YR 6, DOWNSHIFT, N Solve for FV = 35,821.5689, or approximately $35,822.

(Martin Case Study) Calculate the Martins' savings from refinancing their home loan over the first year of the new loan. Assume they refinance with a 30-year loan. The result is

$7,716.72. Current monthly mortgage payment—$1,829.50 Mortgage payment after refinancing—$1,186.44 END Mode 12, DOWNSHIFT, P/YR DOWNSHIFT, C ALL 197888, PV 6, I/YR 30, DOWNSHIFT, N = 360 Solve for PV = -1,186.4385, or $1,186.44 Monthly savings—643.06 × 12 Savings over the first year of the loan—$7,716.72 (Do not factor in closing costs because they are being paid from separate funds.)

Which of the following statements regarding open-ended and closed-ended questions is CORRECT? 1 Planners should use as many open-ended questions as possible when developing client goals and expectations. 2 Closed-ended questions facilitate effective communication between the client and planner because they require the client to answer in her own words.

1 Planners should use as many open-ended questions as possible when developing client goals and expectations. These types of questions require clients to answer in their own words. Open-ended questions facilitate effective communication between clients and planners. Statement II is incorrect. Closed-ended elicit "yes" or "no" answers, and this can restrict communication.

*** To which of the following groups do the CFP Board Fitness Standards apply? 1 CFP® candidates 2 A CFP® practitioner acting in a supervisory capacity 3 Respondents in a CFP Board investigation 4 Professionals Eligible for Reinstatement

1, 4 The Fitness Standards apply to CFP® candidates and Professionals Eligible for Reinstatement (PER). PER includes both individuals who are not currently certified but have been certified by CFP Board in the past and individuals are eligible to reinstate their certification without being required to pass the current CFP® Certification Examination.

To properly use the CFP® marks on documents or marketing materials, certain guidelines must be followed. Identify the items that are required when the words CERTIFIED FINANCIAL PLANNER™ are used. 1 Always use capital letters or small cap font. 2 Always use the ™ symbol. 3 Always associate with the CFP Board. 4 Always use with one of CFP Board's approved nouns ("certificant", "professional", "practitioner", "certification", "mark", or "exam") unless directly following the name of the individual certified by CFP Board.

1,2,4 If the words 'CERTIFIED FINANCIAL PLANNER' are used, they must be presented in capital letters or small cap font followed by a ™ because the words are subject to trademark law. Statement III is incorrect, the words must always associate with the individual(s) certified by CFP Board. 'CERTIFIED FINANCIAL PLANNER' must always be used with one of CFP Board's approved nouns ("certificant", "professional", "practitioner", "certification", "mark" or "exam") unless directly following the name of the individual certified by CFP Board.

Which of the following are representative of a kinesthetic learning style? 1 He prefers to learn by using a hands-on approach. 2 He responds well to graphs, charts, and visual presentations. 3 He retains information by hearing or speaking. 4 He prefers his goals and objectives to be presented as a to-do-list in bullet form.

1,4 Statement I is correct because people who prefer to learn through a hands-on approach have a kinesthetic learning style. Statement II is incorrect because people who respond well to graphs, charts, and visual presentations have a visual learning style. Statement III is incorrect because this is characteristic of an auditory learning style. Statement IV is correct because those individuals who prefer goals and objectives to be presented in bullet form exhibit a kinesthetic learning style.

*** Wendy, a CFP® professional, is meeting with her client, Chrissy, to discuss her goals. Chrissy advises Wendy that one of her goals is to take an expensive hiking trip in the Alps, something her mother and grandfather had done when they were young. She'd like to keep this tradition in her family. Wendy believes that, because of Chrissy's financial circumstances, this is not the best use of Chrissy's money. What should Wendy do to best serve her client, Chrissy? 1 Wendy should understand that expensive trips like hiking in the Alps is an important family tradition to Chrissy. 2 Wendy should consider the trip as one of Chrissy's goals and explain how such a goal would impact her overall financial plan. 3 Wendy should share her opinion with Chrissy and persuade her to abandon this as one of her goals. 4 Because Wendy, as Chrissy's financial planner, does not feel taking the trip is a wise financial decision, she should not include it in Chrissy's financial plan as a goal.

1-2 Wendy should respect Chrissy's wishes and understand that such a trip is part of a family tradition. She should consider the trip as one of Chrissy's goals; however, she should also let her know how such a goal would impact Chrissy's overall financial plan. Wendy should not share her opinion with Chrissy or try to persuade her to abandon it as one of her goals.

The preamble of the Code of Ethics and Standards of Conduct describes the value of upholding aspirational standards and incorporating them into everyday financial planning practice. Identify the purpose(s) of the Code and Standards contained in the preamble. 1 To benefit and protect the public 2 To advance financial planning as a distinct and valuable profession 3 To hold all CFP® professionals to high standards of competency and ethics 4 To investigate, review, consider recommendations and issue final decisions on allegations of violations and/or non-compliance

1-3 Statement IV is incorrect; the CFP Board's Disciplinary and Ethics Commission is responsible for investigating, reviewing, considering recommendations and issuing final decisions on allegations of violations and/or noncompliance.

Whitney is designing her new email template which will reflect her recent certification as a CFP® professional. Under her name, she has identified herself as a CFP™. In a tagline at the bottom of the template, she advertises herself as a "CFP® expert." According to CFP Board's guidelines regarding how the CFP®" marks may be used, which of the following is CORRECT? 1 Whitney has appropriately identified herself as a CFP™. 2 Advertising herself as a CFP® expert is prohibited by CFP Board.

2 Although, Whitney can identify herself as a CFP®, it must be done behind her name, not under it. If Whitney wants to identify herself under her name, she should do so as a ™. Statement II is correct. "CFP®" must always be used with one of CFP Board's approved nouns ("certificant", "professional", "practitioner", "certification", "mark", or "exam") unless directly following the name of the individual certified by CFP Board. Here, it can be used alone.

Charlie wants to sell financial products as a career. In addition to the Series 63 registration required by his state, what other license must he have to sell both variable life insurance and mutual funds? 1 FINRA Series 65 2 FINRA Series 66 3 FINRA Series 6 or 7 4 State variable insurance license

3-4 A planner who wishes to sell variable contracts, such as variable life insurance or variable annuities, must hold a FINRA Series 6 or Series 7 (and, in most states, a Series 63 registration) as well as that particular state's variable insurance license. A Series 65 license entitles the holder to provide investment advice to clients within the holder's primary state of residence. The Series 63 and Series 65 license requirements are combined in the Series 66 registration. Any individual who holds a Series 66 license may provide investment advice and sell securities to any client in any state.

Which of these statements regarding the classical economics approach to financial counseling is CORRECT? 1 This approach features the use of a SWOT analysis. 2 This approach is based on the use of psychoanalytic theory such as Freudian or Gestalt theory. 3 Clients choose among alternatives based on objectively defined cost-benefit and risk-return tradeoffs. 4 This approach believes that increasing financial resources or reducing financial expenditures results in improved financial outcomes.

3-4 Statement I is incorrect; the strategic management approach features the use of a SWOT analysis. Statement II is incorrect; the financial counseling approach that is based on the use of psychoanalytic theory such as Freudian or Gestalt theory is the psychoanalytic approach.

As a financial planner you must be familiar with several different economic and interest rates, and how to use them in calculations that are important to your clients. Assume the following rates: Prime rate 5.5% Interest rate − Investment A 8.0% Inflation rate 2.5% Discount rate 2.0% Considering all the interest rates above, calculate the inflation-adjusted return for investment A.

5.37%. [(1.08 ÷ 1.025) − 1] x 100 = 5.3659, rounded to 5.37% This can also be calculated using the method below. 1.025, INPUT 1.08, DOWNSHIFT, % CHG Solve for I/YR = 5.37%

Charley invested $100,000 12 years ago. Today, it is valued at $250,000. What is the rate of return for Charley's investment?

7.93%. This requires an interest rate calculation. The keystrokes on the HP 10bII/HP 10bII+ are as follows: END Mode 1, DOWNSHIFT, P/YR DOWNSHIFT, C ALL 100000, +/-, PV 250000, FV 12, DOWNSHIFT, N Solve for I/YR = 7.9348, or 7.93%.

According to CFP Board Code of Ethics, Connor, a CFP® professional, is responsible for which of the following? 1 Acting in the client's best interests 2 Avoiding or disclosing and managing conflicts of interest 3 Acting with honesty, integrity, competence, and diligence 4 Maintaining the confidentiality and protect the privacy of client information

All CFP® professional must Act with honesty, integrity, competence, and diligence. Act in the client's best interests. Exercise due care. Avoid or disclose and manage conflicts of interest. Maintain the confidentiality and protect the privacy of client information. Act in a manner that reflects positively on the financial planning profession and CFP® certification.

Amanda, a CFP® professional, is enjoying an afternoon at her son's school playground when she is approached by Tracy, a fellow parent and teacher at a local elementary school. Their conversation covers specifics about Tracy's personal and financial situation, with emphasis on a recommended investment strategy for her 403(b) account. Amanda is aware that she is providing Financial Advice to Tracy. Identify the correct application of rules from the Code and Standards based on Amanda and Tracy's interaction.

Although only Financial Advice has been determined, Amanda must uphold her Fiduciary Duty and must follow the Code of Ethics. As a CFP® professional, Amanda is obligated to uphold the Code of Ethics. In addition, since Financial Advice is occurring, Amanda must act as a Fiduciary, in the best interest of Tracy. The introduction of the Code and Standards specifically states "The Code of Ethics applies at all times, and sets forth principles that guide the behavior of CFP® professionals, with elaboration provided in the Standards."

*** Paul and Carmen recently moved to the Gulf Coast. Paul is in the Secret Service and would like to live on the beach until he gets his next work assignment in three years. Paul and Carmen want to purchase a home; however, they expect to move for Paul's next assignment. Which of the following mortgages is best for Paul and Carmen if they want to keep their monthly mortgage payments to a minimum?

An adjustable rate mortgage with an interest rate cap Given Paul and Carmen's relatively short time in their new home, an adjustable rate mortgage (ARM) is their best option. A reverse mortgage is a special type of home loan where the payment stream is reversed (that is, the lender pays the homeowner a stream of income secured by the considerable amount of equity in the home).

Ed recently began studying for his master's degree earlier this year on a half-time basis. He has applied for a Pell Grant and a Federal Supplemental Educational Opportunity Grant (FSEOG). Assuming Ed demonstrates financial need, for which of the following is he likely to be approved?

Because he is a graduate student, Ed will not qualify for either grant.

*** Recently, Fallon, an avid shopper, has heard from her friends that an investment in Shoes-2-You stock was a wise idea because the shoes sold are very stylish. Even though Fallon's financial planner has advised her that investing in this stock is a poor decision, she invests in it anyway. Her brother, Stanley, congratulates her on her investment because he feels it is a wise investment. Stanley considers himself to be an expert in investments. Unfortunately, he considers his expertise to be much greater than it actually is. In the past, Stanley has taken credit for any investment decisions that have positive returns but blames the economy when an investment does poorly. Considering Fallon's and Stanley's behavior, which of the following statements is CORRECT?

Fallon's behavior is an example of confirmation bias; Stanley's behavior is representative of overconfidence. Confirmation bias is paying attention to information that supports a preconceived opinion and poorly made decision, while disregarding accurate, unsupportive information. Overconfidence tends to make Stanley believe his level of ability is much higher than what it is. Anchoring is making irrational decisions based on information that should have no influence on the decision at hand. Mental accounting is putting money into separate "accounts" based on the function of these accounts.

(Martin Case Study) Analyze the following scenario. Ryan's neighbor, Dan, asked you to prepare his tax return and financial statements for the previous year. You do this for Dan and meet with him when the documents are completed. During your meeting, you present an investment plan for Dan based on the information he provided for the tax return preparation. Dan thanks you for completing the tax return and financial statements and pays you a fee for your work. Dan, however, tells you he is not interested in an investment plan at this time but may contact you in the future if circumstances change. Assuming you are competent to prepare the tax return and financial statements, did you provide financial planning for Dan based on the facts presented?

No, because Dan did not ask you to prepare, nor did he expect you to create, an investment plan. Dan expected no more from you than the preparation of his tax return and financial statements; he did not request any of the financial planning subject areas be addressed. The fact that Dan paid you a fee for your services, and that Dan is Ryan's neighbor, does not make the assistance you provided to Dan financial planning. In some cases, a client's request for a singular service may rise to the level of financial planning. Depending on the depth and breadth of a service and the intent of the client, as well as the comprehensiveness of information gathered, a service might be considered financial planning.

Which of the following acts provides applicable procedures for issuing an initial public offering (IPO) of securities and addresses the paperwork involved in offering new securities to potential investors?

Securities Act of 1933. In addition, this act specifies which securities are exempt from registration requirements. The 1933 Act also requires the registration of new issues of securities in the primary market.

Over the years, Quinn has made timely payments on four of his credit card accounts, all which have balances near the available credit limits. He did pay off a fifth credit card account, which he had for 20 years, and immediately closed it. Which of the following statements regarding Quinn's credit score is CORRECT? 1 By immediately closing his long-standing account when it was paid off, Quinn likely decreased his credit score. 2 Having four credit card account balances near their available credit limits will in all likelihood adversely affect Quinn's credit score.

both Immediately closing long-standing accounts will likely decrease Quinn's credit score. Keeping account balances near the available credit limit has a negative effect.

Which of the following statements concerning investment advisers is CORRECT? 1 Investment advisers with assets under management of less than $100 million will generally be required to register as an investment adviser with the state in which they maintain clients. 2 In general, investment advisers with $100 million of assets or more under management are required to register with the SEC.

both Investment advisers with assets under management of less than $100 million will generally be required to register as an investment adviser with the state in which they maintain clients. The general rule is that investment advisers with $100 million of assets or more under management must register with the SEC.


Ensembles d'études connexes

Finance 3630: Equity Securities and Markets - Chapter 3 Part 2

View Set

RN Comprehensive Online Practice 2019 B with NGN

View Set

MGMT 470 Test 3 Review (6-10 and case studies)

View Set